Hjælp til HA - VØ

#1| 0

Har siddet og bikset med en opgave i ret lang tid nu her, så håber lidt der en venlig sjæl, der vil hjælpe? :)

Jeg ved så meget at man skal differentiere ift. L. derved kan det ses at holdes K konstant og ændres L, bliver MPL mindre.
Derfor er grænseproduktet MPl aftagende mht. L

Synes dog ikke rigtigt at resultat jeg får giver meget mening, derfor vil jeg gerne have et andet syn på sagen.. ;)



En virksomhed har produktionsfunktionen Q=K^0,5L^0,5, hvor Q er output. K er kapital og L er arbejdskraft.

Vis hvorvidt virksomheden har aftagende grænseprodukt med hensyn til L


02-01-2013 23:58 #2| 0

dQ/dL=(K^0,5)*(0,5L^(-0,5))=(K^0,5)*(0,5*(1/(L^0,5)))

Så altså hvis K holdes konstant, så skal du bare fokusere på 1/(L^0,5). Det led går mod 0 når L går mod uendelig og dermed går hele funktionen dQ/dL mod 0 når L går mod uendelig, altså har produktionsfunktionen Q aftagende grænseprodukt med hensyn til L.

EDIT: Ændret notation. Det er i øvrigt en Cobb-Douglas-produktionsfunktion :)

Redigeret af Yarx d. 03-01-2013 00:08
03-01-2013 16:07 #3| 0
OP

Mange tak for hjælpen.. :)

Ved godt at en Cobb Douglas funktion altid har et aftagende grænseprodukt, var bare irreteret over at jeg ikke kunne finde ud af den.. :)

03-01-2013 16:38 #4| 0

IANAE, men er det rigtigt?

Betyder "grænseprodukt" ikke produktionsforøgelsen for den sidst investerede arbejdskraftenhed? Altså dQ/dL?

I så fald burde man rent matematisk skulle godtgøre, at den dobbeltafledede funktion er negativ for alle L>0 for at vise det ønskede. Ikke at dQ/dL går mod 0 for L gående mod uendelig.

Er det mig, der er i skoven her?

03-01-2013 16:44 #5| 0
OP

Vil du mene det her sådan her den løses Henry?

For det er nemlig det jeg er ude i..

q=L^(1/2)*K^(1/2)

MP_L=1/2*(K^(1/2)/L^(1/2) )

MP_K=1/2*(L^(1/2)/K^(1/2) )

Aftagende marginal produkt på kapital og arbejdskraft.

Skala afkast:

q^'=(λL)^(1/2)*(λK)^(1/2)=λ^(1/2)*L^(1/2)*λ^(1/2)*K^(1/2)
q^'=λ*q

03-01-2013 17:47 #6| 0

Nej, jeg ville gøre flg:

MP_L = Q'(L) = 0,5 * K^(0,5) * L^(-0,5)

MP_L' = Q''(L) = -0,25 * K^(0,5) * L^(-1,5)

Dvs at MP_L' er negativ, idet L og K er positive tal.

Når MP_L' er negativ overalt, er MP_L (grænseproduktet) en aftagende funktion mht L. Hvilket jeg opfatter var det, der skulle vises.

Jeg er ikke økonom, så jeg kan som sagt være i skoven et sted.

03-01-2013 21:03 #7| 0

henry's metode kan man selvfølgelig også bruge. Det kommer bare an på hvordan man har lyst til at argumentere.

← Gå til forumoversigtenGå til toppen ↑
Skriv et svar